LSAT and Law School Admissions Forum

Get expert LSAT preparation and law school admissions advice from PowerScore Test Preparation.

 smile22
  • Posts: 135
  • Joined: Jan 05, 2014
|
#14472
For this question, I diagrammed the stimulus as follows:

public funding justified :arrow: indication of how public will benefit

The stimulus concludes that if the critics are correct, then there would not be public support.
Based on this, I question whether the principle above should be followed considering that there would not be public support for the funding even if there is an indication of how the public will benefit. For this reason, is that why answer E is correct?
 Emily Haney-Caron
PowerScore Staff
  • PowerScore Staff
  • Posts: 577
  • Joined: Jan 12, 2012
|
#14477
Hello,

You did a great job diagramming! The scientists here are basically arguing that the critics are wrong - there's no indication of how the public will benefit, but there is still strong public support, and it should be funded. Therefore, E is correct.
 smile22
  • Posts: 135
  • Joined: Jan 05, 2014
|
#14482
Thank you for the explanation!
 est15
  • Posts: 94
  • Joined: Aug 28, 2013
|
#15943
Why isn't A the right answer? I thought that if the social impact of the new drugs being tested were poorly understood then they shouldn't be brought to the market as quickly.
 Ron Gore
PowerScore Staff
  • PowerScore Staff
  • Posts: 220
  • Joined: May 15, 2013
|
#15970
Hi Est!

Thanks for asking this question, which has a very simple answer (I believe :-D ) .

It appears that you have misread the answer key. Answer choice (A) is the correct answer for this question.

Let me know if I can help further.

Thanks!

Ron
 est15
  • Posts: 94
  • Joined: Aug 28, 2013
|
#15997
Hi Ron, I meant to ask why B isn't the right answer. I thought that if the social impact of the new drugs being tested were poorly understood then they shouldn't be brought to the market as quickly. Sorry about the typo!
 Ron Gore
PowerScore Staff
  • PowerScore Staff
  • Posts: 220
  • Joined: May 15, 2013
|
#16001
Ah! That makes more sense. :-D

The problem with answer choice (B) is that it references drugs that are "being tested," but not necessarily being marketed. Since we do not know whether the drugs referenced in answer choice (B) are being brought to the marketplace, (B) has no impact on the conclusion.

Thanks,

Ron
 elewis10
  • Posts: 21
  • Joined: Sep 02, 2017
|
#45137
Ron Gore wrote:Hi Est!

Thanks for asking this question, which has a very simple answer (I believe :-D ) .

It appears that you have misread the answer key. Answer choice (A) is the correct answer for this question.

Let me know if I can help further.

Thanks!

Ron

I thought E was the right answer?
 Malila Robinson
PowerScore Staff
  • PowerScore Staff
  • Posts: 296
  • Joined: Feb 01, 2018
|
#45209
Hi elewis10,
Sorry for the confusion. E is the correct answer.
-Malila
 chian9010
  • Posts: 81
  • Joined: Jun 08, 2018
|
#59002
Could anyone please explain why A is not the correct answer?

Get the most out of your LSAT Prep Plus subscription.

Analyze and track your performance with our Testing and Analytics Package.